Una cierta amplitud de dispersión de gluones

Estoy atascado con este proceso de calcular la amplitud de dispersión a nivel de árbol de dos gluones de impulso de helicidad positiva (+), digamos pag 1 y pag 2 dispersándose en dos gluones de helicidad negativa (-) con impulso pag 3 y pag 4 .

esto es aparentemente 0 para el diagrama donde uno ve este proceso como dos amplitudes de 3 gluones con un gluón que se propaga (de digamos momento pag ) y pag 1 y pag 2 están unidos uno a cada uno de los dos 3 amplitudes de gluones. Quiero ser capaz de probar esta desaparición.

Entonces deja pag 2 + estar con pag y pag 3 y descansa sobre los otros 3 vértices de gluones.

Estoy trabajando en el formalismo despojado de color. Sean los índices de Lorentz ρ , σ para el gluón que se propaga. Y para los gluones externos pag 1 + , pag 2 + , pag 3 , pag 4 dejar v , λ , β , m respectivamente sus índices de Lorentz. Sean los vectores auxiliares elegidos para especificar las polarizaciones de estos gluones externos pag 4 , pag 4 , pag 1 , pag 1 respectivamente. Entonces, las "funciones de onda" de estos cuatro gluones se denotan como, ϵ + / ( pag , norte ) dónde pag representa su impulso y norte su vector auxiliar y en el formalismo espinor-helicidad se escribiría,

  1. ϵ m + ( pag , norte ) = < norte | γ m | pag ] 2 < norte | pag >

  2. ϵ m ( pag , norte ) = [ norte | γ m | pag > 2 [ pag | norte ]

Por lo tanto, pensaría que esta amplitud está dada por,

ϵ m ( pag 4 , pag 1 ) ϵ v + ( pag 1 , pag 4 ) ϵ λ + ( pag 2 , pag 4 ) ϵ β ( pag 3 , pag 1 ) ( i gramo 2 ) 2 × { η m v ( pag 4 pag 1 ) ρ + η v ρ ( pag 1 pag ) m + η ρ m ( pag pag 4 ) v } ( i η ρ σ pag 2 ) { η λ β ( pag 2 pag 3 ) σ + η β σ ( pag 3 pag ) λ + η σ λ ( pag pag 2 ) β }

Se observa lo siguiente,

  1. ϵ m ( k 1 , norte ) . ϵ m ( k 2 , norte ) = ϵ m + ( k 1 , norte ) . ϵ + m ( k 2 , norte ) = 0

  2. ϵ m + ( k 1 , norte 1 ) . ϵ m ( k 2 , norte 2 ) ( 1 d k 2 norte 1 ) ( 1 d k 1 , norte 2 )

Usando lo anterior, se ve que en la amplitud dada, el único término que no se desvanece que queda es (hasta algunos prefactores),

ϵ m ( pag 4 , pag 1 ) ϵ v + ( pag 1 , pag 4 ) ϵ λ + ( pag 2 , pag 4 ) ϵ β ( pag 3 , pag 1 ) { η σ v ( pag 1 pag ) m + η σ m ( pag pag 4 ) v } × { η λ β ( pag 2 pag 3 ) σ }

(..el que es el producto de los dos últimos términos del factor del primer vértice (contraído con el índice del propagador) y el primer término del factor del segundo vértice..}

  • ¿Por qué está este término por encima de cero? (... la única forma en que todo el diagrama puede desaparecer...)
¡No puedo ver por qué mi última ecuación se ha distorsionado! Sería genial si alguien puede rectificar eso.
No es físico hablar de la desaparición de un diagrama. Los diagramas no son invariantes de calibre, las amplitudes sí lo son. Solo tiene sentido cuando especifica un indicador, es decir, una selección de vectores de polarización. (No he leído ninguno de sus detalles, pero tal vez pensar en el hecho general le sea útil).
@Matt Reece Gracias por tu comentario. Supongo que el punto que estás planteando ahora se aclara conmigo especificando los vectores de polarización ( ϵ + / ( pag , norte ) ) que tenía en mente. He añadido sus expresiones explícitas. Hay 3 diagramas ordenados por colores que contribuyen a este proceso de dispersión, aquel cuya expresión he escrito arriba es uno de los dos de esos 3 que se desvanecen. Sería muy útil si pudiera explicar por qué la expresión final de la mía es cero (¡o si hay algo mal en la expresión misma!)
Elegir un calibre significa elegir un espinor de referencia | norte (o | norte ] , dependiendo de la helicidad). A menudo, desea elegir diferentes partículas para que tengan el mismo espinor de referencia, o tal vez el espinor de referencia provenga de otro impulso en el proceso, para hacer que la mayor cantidad posible de términos sea cero.
@Matt Reece Eso es exactamente lo que aclaré en el tercer párrafo. Mi elección para los espinores de referencia corresponde a momentos pag 4 , pag 4 , pag 1 , pag 1 para partículas con momento y helicidad pag 1 + , pag 2 + , pag 3 , pag 4 respectivamente. Supongo que eso aclara el significado completo de la amplitud que he escrito.

Respuestas (1)

Estaba tratando de empujarte en la dirección correcta, pero aquí está el cálculo explícito. Concéntrese en el vértice donde se encuentran los gluones 1 y 4. ahí tienes un factor ϵ ( pag 1 ) v ϵ ( pag 4 ) m ( η m v ( pag 4 pag 1 ) ρ + η v ρ ( pag 1 pag ) m + η ρ m ( pag pag 4 ) v ) . Pero, por construcción, ϵ ( pag 1 ) ϵ ( pag 4 ) = 0 , entonces el η m v término se desvanece. En el segundo término, usamos pag = pag 1 pag 4 notar que tenemos un factor ( 2 pag 1 pag 4 ) ϵ ( pag 4 ) . Pero ϵ ( pag 4 ) pag 4 = 0 porque los bosones de calibre son transversales, mientras que pag 1 ϵ ( pag 4 ) = 0 por su elección del espinor de referencia para el gluón 4. Entonces, el segundo término es cero. El último término es cero de manera análoga. Entonces este vértice es cero y no necesitas pensar en el resto del diagrama.